Disuguaglianza apparentemente innocua

Polinomi, disuguaglianze, numeri complessi, ...
Avatar utente
Drago96
Messaggi: 1147
Iscritto il: 14 mar 2011, 16:57
Località: Provincia di Torino
Contatta:

Re: Disuguaglianza apparentemente innocua

Messaggio da Drago96 »

Ti sei perso un $*(a+b+c) $, ma forse si aggiusta...
Quello che non mi torna è invece la disuguaglianza sulla parabola: il vertice si trova in 1, quindi non capisco bene da dove spunta fuori l'espressione a destra...
Imagination is more important than knowledge. For knowledge is limited, whereas imagination embraces the entire world, stimulating progress, giving birth to evolution (A. Einstein)
Avatar utente
gpzes
Messaggi: 173
Iscritto il: 01 gen 1970, 01:00
Contatta:

Re: Disuguaglianza apparentemente innocua

Messaggio da gpzes »

:oops: :oops: ehh si..ma temo aver fatto casini... :oops: :oops:
Avatar utente
Kfp
Messaggi: 188
Iscritto il: 20 mag 2012, 19:17
Località: Brescia

Re: Disuguaglianza apparentemente innocua

Messaggio da Kfp »

HCP16 ha scritto:Beh invece non dovrebbe fare parte di una soluzione completa, la richiesta del problema è uguale a: "dimostra che il minimo valore di f è -3" , non capisco perché tu consideri completezza anche trovare il limite superiore, è semplicemente andare oltre le richieste del problema, ragione per cui non ho risolto le equazioni ''miste'' che hai citato alle 13.07, solo perché non è richiesto dal problema quindi perché cruciarsi con più calcoli?
Provando a chiarirti le idee in maniera cordiale (e alquanto dissimile dalla tua): nel dimostrare qualcosa coi moltiplicatori di Lagrange tu devi controllare che TUTTI i punti che trovi stiano sopra al valore che vuoi dimostrare essere minimo (-3), dato che a priori scartandoli come fai tu nulla ci assicura che uno di essi non valga, che ne so, -100000000000 (oltre a verificare che non ci siano punti sul bordo che abbassano il minimo). Tu hai solo trovato che uno dei punti in questione il valore che ti interessa è proprio -3, ma perchè questa sia una dimostrazione vera devi anche crucciarti con tutti gli altri calcoli (poi grazie che la tesi è vera e ti verrà che quello è proprio il valore minimo, ma senza l'altro pezzo non hai dimostrato nulla).
"Signora, lei sì che ha le palle, mica come quella checca di suo figlio"

"La zuppa magica dedicata a te Gianluca"

"È "iamo", non rompere i coglioni"
Dandav
Messaggi: 24
Iscritto il: 12 ago 2011, 11:05

Re: Disuguaglianza apparentemente innocua

Messaggio da Dandav »

...alquanto dissimile...
Wooo...
Avatar utente
gpzes
Messaggi: 173
Iscritto il: 01 gen 1970, 01:00
Contatta:

Re: Disuguaglianza apparentemente innocua

Messaggio da gpzes »

L’idea è cercare di applicare la disuguaglianza di Jensen.
1° Fatto: consideriamo $f(x)={{x}^{3}}-2{{x}^{2}}$. La funzione è convessa per $x\ge 2/3$.

2° Fatto: se$abc=1$, allora $1\le (a+b+c)/3$

Poiché ho valore dentro intervallo di convessità, per Jensen, si ha
$f((a+b+c)/3)\le \left( f(a)+f(b)+f(c) \right)/3$.
Posto $A=(a+b+c)/3$e sostituendo, abbiamo ${{A}^{3}}-2{{A}^{2}}\le \left( {{a}^{3}}+{{b}^{3}}+{{c}^{3}}-(2{{a}^{2}}+{{b}^{2}}+{{c}^{2}}) \right)/3$
e, in definitiva, $3({{A}^{3}}-2\cdot {{A}^{2}})\le \left( {{a}^{3}}+{{b}^{3}}+{{c}^{3}}-(2{{a}^{2}}+{{b}^{2}}+{{c}^{2}}) \right)$.
Allora, $3{{A}^{2}}(A-2)\le \left( {{a}^{3}}+{{b}^{3}}+{{c}^{3}}-(2{{a}^{2}}+{{b}^{2}}+{{c}^{2}}) \right)$.
Ma $3{{A}^{2}}(A-2)\ge -3$, perché $A\ge 1$.
Aggiungendo un 3 ad entrambi i membri della disuguaglianza otteniamo
$0\le {{a}^{3}}+{{b}^{3}}+{{c}^{3}}-(2{{a}^{2}}+{{b}^{2}}+{{c}^{2}})+3$ e quindi la tesi.

..mahh :oops: non so mai... :oops:
HCP16
Messaggi: 21
Iscritto il: 23 ago 2014, 18:38

Re: Disuguaglianza apparentemente innocua

Messaggio da HCP16 »

Altra soluzione:
da AM-GM abbiamo $ a^2b+b^2c+c^2a\ge{3abc} $ , sommandola con l'ipotesi e semplificando rimane da dimostrare che $ a^3+b^3+c^3\ge{a^2b+b^2c+c^2a} $ , ma è vera per Muirhead poichè $ (3,0,0) $ maggiorizza $ (2,1,0) $.
fph
Site Admin
Messaggi: 3957
Iscritto il: 01 gen 1970, 01:00
Località: in giro
Contatta:

Re: Disuguaglianza apparentemente innocua

Messaggio da fph »

Mi son perso che fine fanno i termini di grado 2 nel tuo conto. In ogni caso per finire non puoi usare Muirhead perché $a^2b+b^2c+c^2a$ è una somma ciclica, non simmetrica. Quella disuguaglianza è vera, ma va dimostrata in un altro modo (hint:
Testo nascosto:
riarrangiamento
)
--federico
[tex]\frac1{\sqrt2}\bigl(\left|\text{loves me}\right\rangle+\left|\text{loves me not}\right\rangle\bigr)[/tex]
HCP16
Messaggi: 21
Iscritto il: 23 ago 2014, 18:38

Re: Disuguaglianza apparentemente innocua

Messaggio da HCP16 »

Risposta a fhp: intanto scusa perché sono andato direttamente alla pagina 3 senza riguardare il testo e me lo ricordavo male, comunque oltre al riarrangiamento si può fare anche così credo: da AM-GM--> $ a^3+a^3+b^3\ge{3a^2b} $ ; $ b^3+b^3+c^3\ge{3b^2c} $ ; $ c^3+c^3+a^3\ge{3c^2a} $ , sommandole tutte e dividendo per 3 avremmo la tesi (che è circa come quando non si vuole andare di triste bunching e si fanno tutte le AM-GM si sommano e si mette a posto). Tornando alla nostra disuguaglianza: da AM-GM abbiamo che $ a^3+b^3+c^3\ge3abc $ , che sommata con l'ipotesi da $ 2(a^3+b^3+c^3)+3abc\ge{2(a^2+b^2+c^2)+3abc} $, quindi $ a^3+b^3+c^3\ge{a^2+b^2+c^2} $ con sempre $ abc=1 $, che scusate ma non so concludere formalmente :?
fph
Site Admin
Messaggi: 3957
Iscritto il: 01 gen 1970, 01:00
Località: in giro
Contatta:

Re: Disuguaglianza apparentemente innocua

Messaggio da fph »

Non puoi sommare una disuguaglianza all'ipotesi in questo modo, perché "va nel verso sbagliato"; se provi a scrivere quella cosa come una catena unica di disuguaglianze (o a chiederti quale teorema/proprietà stai applicando) ti incarti. Vedila così: voglio dimostrare $1>2$; io so che $10>5$, quindi lo sommo a entrambi i membri e mi resta da dimostrare $11>7$, che è vero. :?:
--federico
[tex]\frac1{\sqrt2}\bigl(\left|\text{loves me}\right\rangle+\left|\text{loves me not}\right\rangle\bigr)[/tex]
HCP16
Messaggi: 21
Iscritto il: 23 ago 2014, 18:38

Re: Disuguaglianza apparentemente innocua

Messaggio da HCP16 »

Giusto hai ragione scusa, comunque come si concluderebbe in un bel modo $ a^3+b^3+c^3\ge{a^2+b^2+c^2} $ con $ abc=1 $ ?
fph
Site Admin
Messaggi: 3957
Iscritto il: 01 gen 1970, 01:00
Località: in giro
Contatta:

Re: Disuguaglianza apparentemente innocua

Messaggio da fph »

Ci sono forse metodi più di fioretto, ma ti dico quello industriale che secondo me è il modo in cui conviene pensare le disuguaglianze a un certo livello. Prima di tutto omogeneizza, viene $a^3+b^3+c^3 = (a^2+b^2+c^2)\sqrt[3]{abc}$. A questo punto, noti che il LHS ha tutti gli esponenti più "concentrati" del RHS, quindi non può che venire. Possibilità: poni $x=a^{1/3}$ e cicliche e usi Muirhead, oppure fai come hai fatto tu prima e stimi con AM-GM pesata $a^{7/3}b^{1/3}c^{1/3} \leq \frac{7a^3 + b^3 + c^3}{9}$, oppure applichi riarrangiamento a tre specie sulle triplette $(a^{7/3},b^{7/3},c^{7/3})$, $(a^{1/3},b^{1/3},c^{1/3})$, $(a^{1/3},b^{1/3},c^{1/3})$... insomma, quando hai tutti gli esponenti più concentrati da un altro hai vinto. È per questo che la cosa che rompe le scatole nella disuguaglianza iniziale è il termine $abc$ nel LHS. Non puoi farlo sparire in modo indolore o trasformarlo in un $a^3+b^3+c^3$. L'unica delle disuguaglianze note che ti permette di gestire un termine "debole" nel lato grande è Schur; è per questo che questo problema vi deve far pensare a Schur. Dovete chiedervi (1) riesco ad applicare Schur, oppure (2) quali dimostrazioni di Schur so? Riesco a modificarle ed adattarle fino a farle diventare una dimostrazione di questa disuguaglianza?
Disclaimer: non l'ho fatto questo problema, :p ma se me lo trovassi davanti in gara il tentativo n.1 che farei è questo. Ha l'aria medio-difficile.

Quindi, take-home messages:
(1) scrivi e pensa una soluzione in termini di catene di disuguaglianze. In questo modo eviti di fare errori evitabili tipo una disuguaglianza che va nel verso sbagliato.
(2) pensa le disuguaglianze polinomiali in termini di "esponenti concentrati", "termini forti" e "termini deboli".
--federico
[tex]\frac1{\sqrt2}\bigl(\left|\text{loves me}\right\rangle+\left|\text{loves me not}\right\rangle\bigr)[/tex]
HCP16
Messaggi: 21
Iscritto il: 23 ago 2014, 18:38

Re: Disuguaglianza apparentemente innocua

Messaggio da HCP16 »

Guarda grazie mille per la risposta, veramente completa e molto utile , dopo proverò a pensarci un po' con Schur e sue dimostrazioni :)
Avatar utente
Tess
Messaggi: 272
Iscritto il: 15 set 2009, 14:20
Località: Maserada s. P.

Re: Disuguaglianza apparentemente innocua

Messaggio da Tess »

Insomma, il propositore alla fine ha un'idea carina di come risolvere il problema? Credo sia arrivato il momento di dare almeno qualche indizio...
Avatar utente
gpzes
Messaggi: 173
Iscritto il: 01 gen 1970, 01:00
Contatta:

Re: Disuguaglianza apparentemente innocua

Messaggio da gpzes »

gpzes ha scritto:L’idea è cercare di applicare la disuguaglianza di Jensen.
1° Fatto: consideriamo $f(x)={{x}^{3}}-2{{x}^{2}}$. La funzione è convessa per $x\ge 2/3$.

2° Fatto: se$abc=1$, allora $1\le (a+b+c)/3$

Poiché ho valore dentro intervallo di convessità, per Jensen, si ha
$f((a+b+c)/3)\le \left( f(a)+f(b)+f(c) \right)/3$.
Posto $A=(a+b+c)/3$e sostituendo, abbiamo ${{A}^{3}}-2{{A}^{2}}\le \left( {{a}^{3}}+{{b}^{3}}+{{c}^{3}}-(2{{a}^{2}}+{{b}^{2}}+{{c}^{2}}) \right)/3$
e, in definitiva, $3({{A}^{3}}-2\cdot {{A}^{2}})\le \left( {{a}^{3}}+{{b}^{3}}+{{c}^{3}}-(2{{a}^{2}}+{{b}^{2}}+{{c}^{2}}) \right)$.
Allora, $3{{A}^{2}}(A-2)\le \left( {{a}^{3}}+{{b}^{3}}+{{c}^{3}}-(2{{a}^{2}}+{{b}^{2}}+{{c}^{2}}) \right)$.
Ma $3{{A}^{2}}(A-2)\ge -3$, perché $A\ge 1$.
Aggiungendo un 3 ad entrambi i membri della disuguaglianza otteniamo
$0\le {{a}^{3}}+{{b}^{3}}+{{c}^{3}}-(2{{a}^{2}}+{{b}^{2}}+{{c}^{2}})+3$ e quindi la tesi.

..mahh :oops: non so mai... :oops:
..
Avatar utente
Tess
Messaggi: 272
Iscritto il: 15 set 2009, 14:20
Località: Maserada s. P.

Re: Disuguaglianza apparentemente innocua

Messaggio da Tess »

E se invece ci fossero uno o 2 valori nella zona concava? In quel caso Jensen non puoi applicarlo (o almeno non puoi applicarlo a tutte le variabili)...
Rispondi